Twelve books cost £51.60. How many books would I get for £193.50?

Answers

Answer 1

Answer:

45 books

Step-by-step explanation:

12=51.60

how about 193.50

193.50 x 12=2322 / 51.60=45 books    

Answer 2
Answer:45 books

Solution,

Books Price

12 51.60

X(suppose) 193.50

In case of Direct proportion,

12/X=51.60/193.50

or,51.60x=2322( cross multiplication)

or,X=2322/51.60

X=45

Hope it helps

Good luck on your assignment


Related Questions

A box contains five keys, only one of which will open a lock. Keys are randomly selected and tried, one at a time, until the lock is opened (keys that do not work are discarded before another is tried). Let Y be the number of the trial on which the lock is opened. Find the probability function for Y.

Answers

The "density of probability" is represented by the Y-axis in the normal distribution.It indicates the likelihood of finding values close to comparable places on the X-axis.

Find the probability f ?

Y is the number of trial on which lock is opend so, P(Y=y) be the probability that key opens  the lock on y th trial Lets draw first key

p(1 st key opens the lock) =1-1/5= 4/5

Hence p(1 st key does not open the lock  = 1- 1/5 =4/5

Since  p (Lock is opened on 1 st triual ) = p(1st key opens the lock )

Hence , P(Y =2 ) 1/4 .4/5 = 1/5

p( y = 4) = 1/5

p(y = 5 ) =1/5

To learn more about probability function refer

https://brainly.com/question/28021875

#SPJ2

The radius of a circle is 6.5cm What is the diameter​

Answers

The diameter of a circle is twice the radius.

Diameter = radius x 2

                = 6.5 x 2

                = 13 cm

Answer:

13

Step-by-step explanation:

diameter is two times the radius

6.5 x 2 = 13

ASAP! GIVING BRAINLIEST! Please read the question THEN answer CORRECTLY! NO guessing. I say no guessing because people usually guess on my questions.

Answers

Answer:

A

Step-by-step explanation:

(f-g)(x) means that the 2 functions are being subtracted.

[tex]3^{x}[/tex] +10x -(5x-3) =[tex]3^{x}[/tex] +10x-5x+3

simplify!

[tex]3^{x}[/tex] +5x+3

the answer is A

If eggs cost $3 per dozen, how much would 8 eggs cost?

Answers

Answer:

$2

Step-by-step explanation:

Because 3/12 is 0.25 and then you multiply it by 8 to get 2.

The solution of the system of equations Ax+y=5 and Ax+By=20 is (2,-3). What are the values of A and B

Answers

Answer:

A = 4B = -4

Step-by-step explanation:

Put the values of x and y in the equations and solve the resulting system.

  A(2) +(-3) = 5

  A(2) +B(-3) = 20

__

The first equation tells the value of A:

  2A = 8 . . . . add 3

  A = 4 . . . . . .divide by 2

This and the second equation tells the value of B:

  (4)(2) -3B = 20

  -3B = 12 . . . . . subtract 8

  B = -4 . . . . . . . divide by -3

The values of A and B are 4 and -4, respectively.

Explain how you will solve the pair of equations by substitution.show all steps and write the solution in (x,y) form.

Answers

Answer:

(-2, -3)

Step-by-step explanation:

The question is incomplete. Here is the complete question.

Consider the following pair of equations:

y = 3x + 3 ... 1

y = x − 1 ... 2

Explain how you will solve the pair of equations by substitution. Show all the steps and write the solution in (x, y) form .

From both equations, we will substitute y = 3x+3 in equation 1 into 2 to have;

3x+3 = x-1

collecting the like terms;

3x-x = -1-3

2x = -4

Dividing both sides by 2 will give;

2x/2 = -4/2

x = -2

Substituting x = -2 into equation 1;

y = 3x+3

y = 3(-2)+3

y = -6+3

y = -3

The solution in (x, y) form is (-2, -3)

The red function was transformed into the blue function. Which transformations have occurred?

Answers

Answer:

Translated 3 units down 2 units left.

Step-by-step explanation:

It looks like the functions are the same, so if you see, it was translated 3 units down and 2 units left.

What’s the correct answer for this question?

Answers

Answer:

E : x=12

Step-by-step explanation:

ACCORDING TO THE THEOREM

(PD)(PC) = (AP)(PB)

8(3+x)=10x

24+8x=10x

24=10x-8x

24 = 2x

x = 12

Answer:

12

Step-by-step explanation:

PD * PC = AP * PB

= 8 * (3 + x) = 10 * x

= 24 + 8x = 10x

= 10x - 8x = 24

= 2x = 24

= 12

The balance on Taylor's credit card is $2000 it has an interest rate of 12.5% she wants to compare the difference between paying $75 and $100 of the monthly balance how much does she save in interest and fees if she pays $100 instead of $75?

Answers

Answer:

so she saved 82 cents in the interest the month after

Step-by-step explanation:

case 1: payment is $75

interest on 2000 = 0.125/12×2000 = $20.83

so the actual repayment on the balance = (75-20.83) = $54.17

therefore,balance =$(2000-54.7)=$1945.83

interest in the next month = $20.27

case 2: payment is $100

interest on 2000 is still $20.83

repayment = $79.73

balance = $1920.27

interest in the next month = 20.01

so she saved 82 cents in the interest the month after

Answer:

$101

Step-by-step explanation:

Check link explanation.

Cuanta cartulina se necesita para construir un prisma rectangular para cuya base mide 10cm y tiene 6cm de altura

Answers

Answer:

i dont understand the language

Step-by-step explanation:

Mr Smith had four daughters.Each daughter has four brothers.How many children does Mr Smith have

Answers

Mr. Smith has 20 children.

Starting off, you already know he has 4 daughters and for every daughter there are 4 sons.

So, we take the 4 daughters and add 4 to each one:

1 daughter + 4 sons = 5 children

We complete that equation 4 times for each daughter, and we end up with 20 children as our answer.

Hope this helps you out a bit! Stay safe and stay healthy! :D

00:00
Muriel has been a member of the Solaris Gym for 372 days,
Ben has been a member for 1 year, 2 weeks, 3 days.
Part A
Who has been a member of the Solaris Gym longer?
Use the drop-down menus to show and explain your answer.
Choose...
has been a member longer because 1 year, 2 weeks, 3 days is
Choose...
than 372 days
00:00
Part B
How much longer? Assume that it is not a leap year. Enter your answer in the box.
days

Answers

Answer:

A: Ben has been a member longer because 1 year, 2 weeks, 3 days is longer than 372 days.

B: 10 days longer

Step-by-step explanation:

1 year, 2 weeks, 3 days is 382 days -> 365 + 14 + 3 = 382

Last week, the price of cherries at the corner deli was $4.99 per pound. This week, cherries at the same deli are on sale at a 15% discount. What is the total price of 3.75 pounds of cherries this week at the deli?

Answers

Answer:

The price for 3.75 pounds of cherries is $17.775.

Step-by-step explanation:

The price las week was $ 4.99 and it got a discount of 15% for this week, this means that the price for this week is 100% of the one prior minus 15%, therefore it is 85% of the last price. We need to calculate the price per pound with this discount as shown below:

[tex]\text{this week} = \text{last week}*\frac{95}{100}\\\\\text{this week} = 4.99*0.95 = 4.74 \text{ per pound}\\[/tex]

Since the price is $4.74 per pound and we want to buy 3.75 pounds, then the total amount we need to pay is:

[tex]\text{total amount} = 3.75*4.74 = 17.775[/tex]

The price for 3.75 pounds of cherries is $17.775.

Please answer this correctly

Answers

Answer:

54 9/10

Step-by-step explanation:

The perimeter is the sum of all the sides

13 1/4 + 12 1/5 + 18 1/4 + 11 1/5

Add up the whole numbers

13+12+18+11 = 54

1/4+1/5+1/4+1/5

2/4+2/5

1/2+2/5

Get a common denominator

1/2*5/5 + 2/5*2/2

5/10 +  4/10

9/10

Add them back together

54 9/10

A car travels at an average speed of 48 miles per hour. How long it take to travel 156 miles

Answers

Answer:

3.25 h = 3 h 15 m

Step-by-step explanation:

156 mi * 1h/48 mi = 3.25 h = 3 h 15 m

Answer: 195 min or 3 hr and 15 min

Step-by-step explanation:

We can set up a proportion to solve this problem.

[tex]\frac{48mi}{60 min} =\frac{156 mi}{x}[/tex]

[tex]48x=156*60[/tex]

[tex]48x=9360[/tex]

[tex]x=195 min[/tex]

We can also write this in terms of hours and minutes.

[tex]\frac{60 min}{1 hr} =\frac{195 min}{x}[/tex]

[tex]60x=195[/tex]

[tex]x=3.25[/tex]

3 hr and 15 min

A shop sells boxes of chocolate. In total there are 252 dark chocolate and 180 milk chocolates.if every box is identical,how many boxes could there be?

Answers

Answer:

12 boxes.

Step-by-step explanation:

We need to find the greatest common factor of 251 and 180:

First find the prime factors :

252 = 2*2*3*3*7

180 =  2*2*3*3*5

So the GCF = 2*2*3*3 = 36.

So id each box is identical, there could be 7 boxes of dark chocolates and 5 boxes of milk chocolates each containing 36 chocolates.

Total number of boxes = 12.

Use a graphing calculator to sketch the graph of the quadratic equation, and then state the domain and range y=2x^2-x+3

Answers

Answer:

For the domain since we have a quadratic function then the domain would be all the real numbers:

[tex] D =[x | x \in R][/tex]

And if we want to find the range we can find the vertex:

[tex] v_x = -\frac{b}{2a}= -\frac{-1}{2*2}= \frac{1}{4}[/tex]

And now we can find th coordinate of y of the vertex like this:

[tex] f(V_x) = 2(\frac{1}{4})^2 -(1/4) +3 =2.875[/tex]

And then the range would be:

[tex] R=[x \geq 2.875][/tex]

Step-by-step explanation:

We have the following function given:

[tex] y = 2x^2 -x +3[/tex]

For this case we can plot the function with a calculator and we got the plot in the figure attached.

For the domain since we have a quadratic function then the domain would be all the real numbers:

[tex] D =[x | x \in R][/tex]

And if we want to find the range we can find the vertex:

[tex] v_x = -\frac{b}{2a}= -\frac{-1}{2*2}= \frac{1}{4}[/tex]

And now we can find th coordinate of y of the vertex like this:

[tex] f(V_x) = 2(\frac{1}{4})^2 -(1/4) +3 =2.875[/tex]

And then the range would be:

[tex] R=[x \geq 2.875][/tex]

A scientist studying water quality measures the lead level in parts per billion (ppb) at each of 49 randomly chosen locations along a water line. Suppose that the lead levels across all the locations on this line are strongly skewed to the right with a mean of 17 ppb and a standard deviation of 14 ppb. Assume that the measurements in the sample are independent. What is the probability that the mean lead level from the sample of 49 measurements T is less than 15 ppb? Choose 1 answer: A) Plæ <15) = 0.02 B) Plū<15) – 0.16 C) Plē <15) 0.30 D) Plö < 15) – 0.44 E) We cannot calculate this probability because the sampling distribution is not normal.

Answers

Complete Question

The complete qustion is shown on the first uploaded image

Answer:

The correct option is B

Step-by-step explanation:

From the question we are told that

      The sample size  is  [tex]n = 49[/tex]

       The mean is  [tex]\mu = 17ppb[/tex]

       The standard deviation is [tex]\sigma = 14 ppb[/tex]

Generally the standard error of this measurement is mathematically represented as

       [tex]\sigma_z = \frac{\sigma}{\sqrt{n} }[/tex]      

substituting values

      [tex]\sigma_{\= x} = \frac{14}{\sqrt{49} }[/tex]  

     [tex]\sigma_{\= x} = 2[/tex]ppb

Now the probability that the mean lead level from the sample of 49 measurements T is less than 15 ppb represented as P(X < 15 )

Next is to find the z value

    [tex]z = \frac{\mu -\sigma }{\sigma_{\= x}}[/tex]

     [tex]z = \frac{15-17}{2}[/tex]

      [tex]z = -1[/tex]

Now checking the z-table for the z-score of  -1 we have  

      [tex]P(X<15) = P(Z < -1 )= 0.16[/tex]

                       

       

     

Using the normal distribution and the central limit theorem, it is found that there is a 0.16 = 16% probability that the mean lead level from the sample of 49 measurements T is less than 15 ppb, hence option B is correct.

In a normal distribution with mean [tex]\mu[/tex] and standard deviation [tex]\sigma[/tex], the z-score of a measure X is given by:

[tex]Z = \frac{X - \mu}{\sigma}[/tex]

It measures how many standard deviations the measure is from the mean.  After finding the z-score, we look at the z-score table and find the p-value associated with this z-score, which is the percentile of X. By the Central Limit Theorem, the sampling distribution of sample means of size n has standard deviation [tex]s = \frac{\sigma}{\sqrt{n}}[/tex], considering a sample size of at least 30 for a skewed variable.

In this problem:

Mean of 17 ppb, hence [tex]\mu = 17[/tex].Standard deviation of 14 ppb, hence [tex]\sigma = 14[/tex].Sample of 49, hence [tex]n = 49, s = \frac{14}{\sqrt{49}} = 2[/tex]

The probability that the mean lead level from the sample of 49 measurements T is less than 15 ppb is the p-value of Z when X = 15, hence:

[tex]Z = \frac{X - \mu}{\sigma}[/tex]

By the Central Limit Theorem

[tex]Z = \frac{X - \mu}{s}[/tex]

[tex]Z = \frac{15 - 17}{2}[/tex]

[tex]Z = -1[/tex]

[tex]Z = -1[/tex] has a p-value of 0.16

0.16 = 16% probability that the mean lead level from the sample of 49 measurements T is less than 15 ppb, hence option B is correct.

For more on the normal distribution and the central limit theorem, you can check https://brainly.com/question/24663213

II. The results of a recent survey indicate that the average new car costs $23,000, with a standard deviation of $3,500. The price of cars is normally distributed. a. What is a Z score for a car with a price of $33,000? b. What is a Z score for a car with a price of $30,000?

Answers

Answer:

a) Z = 2.86

b) Z = 2

Step-by-step explanation:

Z-score:

In a set with mean [tex]\mu[/tex] and standard deviation [tex]\sigma[/tex], the zscore of a measure X is given by:

[tex]Z = \frac{X - \mu}{\sigma}[/tex]

The Z-score measures how many standard deviations the measure is from the mean. After finding the Z-score, we look at the z-score table and find the p-value associated with this z-score. This p-value is the probability that the value of the measure is smaller than X, that is, the percentile of X. Subtracting 1 by the pvalue, we get the probability that the value of the measure is greater than X.

In this question, we have that:

[tex]\mu = 23000, \sigma = 3500[/tex]

a. What is a Z score for a car with a price of $33,000?

This is Z when X = 33000. So

[tex]Z = \frac{X - \mu}{\sigma}[/tex]

[tex]Z = \frac{33000 - 23000}{3500}[/tex]

[tex]Z = 2.86[/tex]

b. What is a Z score for a car with a price of $30,000?

This is Z when X = 30000. So

[tex]Z = \frac{X - \mu}{\sigma}[/tex]

[tex]Z = \frac{30000 - 23000}{3500}[/tex]

[tex]Z = 2[/tex]

The length of time a person takes to decide which shoes to purchase is normally distributed with a mean of 8.21 minutes and a standard deviation of 1.90. Find the probability that a randomly selected individual will take less than 5 minutes to select a shoe purchase. Is this outcome unusual?

Answers

Answer:

4.55% probability that a randomly selected individual will take less than 5 minutes to select a shoe purchase.

Since Z > -2 and Z < 2, this outcome is not considered unusual.

Step-by-step explanation:

Problems of normally distributed samples are solved using the z-score formula.

In a set with mean [tex]\mu[/tex] and standard deviation [tex]\sigma[/tex], the zscore of a measure X is given by:

[tex]Z = \frac{X - \mu}{\sigma}[/tex]

The Z-score measures how many standard deviations the measure is from the mean. After finding the Z-score, we look at the z-score table and find the p-value associated with this z-score. This p-value is the probability that the value of the measure is smaller than X, that is, the percentile of X. Subtracting 1 by the pvalue, we get the probability that the value of the measure is greater than X.

If [tex]Z \leq 2[/tex] or [tex]Z \geq 2[/tex], the outcome X is considered to be unusual.

In this question:

[tex]\mu = 8.21, \sigma = 1.9[/tex]

Find the probability that a randomly selected individual will take less than 5 minutes to select a shoe purchase.

This is the pvalue of Z when X = 5. So

[tex]Z = \frac{X - \mu}{\sigma}[/tex]

[tex]Z = \frac{5 - 8.21}{1.9}[/tex]

[tex]Z = -1.69[/tex]

[tex]Z = -1.69[/tex] has a pvalue of 0.0455.

4.55% probability that a randomly selected individual will take less than 5 minutes to select a shoe purchase.

Since Z > -2 and Z < 2, this outcome is not considered unusual.

Reduce the fraction to lowest terms. Do not use spaces in your answer.

Answers

Answer:

-2x/yz

Step-by-step explanation:

You can cancel out terms using division and properties of exponents. x^a/x^b = x^a-b

Answer is -2x/ya

Hope that helps a bit.

A stock's price fluctuations are approximately normally distributed with a mean of $104.50 and a standard deviation of $23.62. You decide to purchase whenever the price reaches its lowest 10% of values. What is the most you would be willing to pay for the stock?
a) $80.88
b) $74.23
c) $84.62
d) $134.77

Answers

Answer:

[tex]P(z<\frac{a-\mu}{\sigma})=0.10[/tex]

and we can set up the following equation

tex]z=-1.282<\frac{a-104.5}{23.62}[/tex]

And if we solve for a we got

[tex]a=104.5 -1.282*23.62=74.22[/tex]

And the best answer for this case would be:

b) $74.23

Step-by-step explanation:

Let X the random variable that represent the stocks price of a population, and for this case we know the distribution for X is given by:

[tex]X \sim N(104.5,23.62)[/tex]  

Where [tex]\mu=104.5[/tex] and [tex]\sigma=23.62[/tex]

For this part we want to find a value a, such that we satisfy this condition:

[tex]P(X>a)=0.90[/tex]   (a)

[tex]P(X<a)=0.10[/tex]   (b)

As we can see on the figure attached the z value that satisfy the condition with 0.10 of the area on the left and 0.90 of the area on the right it's z=-1.282. On this case P(Z<-1.282)=0.10 and P(z>-1.282)=0.90

If we use condition (b) from previous we have this:

[tex]P(X<a)=P(\frac{X-\mu}{\sigma}<\frac{a-\mu}{\sigma})=0.10[/tex]  

[tex]P(z<\frac{a-\mu}{\sigma})=0.10[/tex]

and we can set up the following equation

tex]z=-1.282<\frac{a-104.5}{23.62}[/tex]

And if we solve for a we got

[tex]a=104.5 -1.282*23.62=74.22[/tex]

And the best answer for this case would be:

b) $74.23

According to Net Market Share, Microsoft's Internet Explorer browser has 53.4% of the global market. A random sample of 70 users was selected. What is the probability that 32 or more from this sample used Internet Explorer as their browser?

Answers

Answer:

Probability that 32 or more from this sample used Internet Explorer as their browser is 0.9015.

Step-by-step explanation:

We are given that according to Net Market Share, Microsoft's Internet Explorer browser has 53.4% of the global market.

A random sample of 70 users was selected.

Let [tex]\hat p[/tex] = sample proportion of users who used Internet Explorer as their browser.

The z score probability distribution for sample proportion is given by;

                            Z  =  [tex]\frac{\hat p-p}{\sqrt{\frac{\hat p(1-\hat p)}{n} } }[/tex]  ~ N(0,1)

where, p = population proportion of users who use internet explorer = 53.4%

           [tex]\hat p[/tex] = sample proportion = [tex]\frac{32}{70}[/tex] = 0.457

           n = sample of users = 70

Now, probability that 32 or more from this sample used Internet Explorer as their browser is given by = P( [tex]\hat p[/tex] [tex]\geq[/tex] 0.457)

      P( [tex]\hat p[/tex] [tex]\geq[/tex] 0.457) = P( [tex]\frac{\hat p-p}{\sqrt{\frac{\hat p(1-\hat p)}{n} } }[/tex] [tex]\geq[/tex] [tex]\frac{0.457-0.534}{\sqrt{\frac{0.457(1-0.457)}{70} } }[/tex] ) = P(Z [tex]\geq[/tex] -1.29)

                            = P(Z [tex]\leq[/tex] 1.29) = 0.9015

The above probability is calculated by looking at the value of x = 1.29 in the z table which has an area of 0.9015.

A manufacturer uses 34 yard of fabric in each skirt.
How many yards of fabric will the manufacturer use in 4 skirts? in 7 skirts? in 9 skirts?

Answers

Answer:

She would use 136 yards in 4 skirts, 238 yards in 7 skirts, and 306 yards in 9 skirts. Hope this helps

Step-by-step explanation:

2x + 3y = 12
Complete the missing value in the solution to the equation.
1,8)

Answers

Answer:

Step-by-step explanation:

[tex]\mathrm{Slope-Intercept\:form\:of}\:2x+3y=12:\quad y=-\frac{2}{3}x+4\\\mathrm{Domain\:of\:}\:-\frac{2}{3}x+4\::\quad \begin{bmatrix}\mathrm{Solution:}\:&\:-\infty \:<x<\infty \\ \:\mathrm{Interval\:Notation:}&\:\left(-\infty \:,\:\infty \:\right)\end{bmatrix}\\\mathrm{Range\:of\:}-\frac{2}{3}x+4:\quad \begin{bmatrix}\mathrm{Solution:}\:&\:-\infty \:<f\left(x\right)<\infty \\ \:\mathrm{Interval\:Notation:}&\:\left(-\infty \:,\:\infty \:\right)\end{bmatrix}[/tex]

[tex]\mathrm{Parity\:of}\:-\frac{2}{3}x+4:\quad \mathrm{Neither\:even\:nor\:odd}\\\mathrm{Axis\:interception\:points\:of}\:-\frac{2}{3}x+4:\quad \mathrm{X\:Intercepts}:\:\left(6,\:0\right),\:\mathrm{Y\:Intercepts}:\:\left(0,\:4\right)\\\mathrm{Inverse\:of}\:-\frac{2}{3}x+4:\quad -\frac{3x-12}{2}\\\mathrm{Slope\:of\:}-\frac{2}{3}x+4:\quad m=-\frac{2}{3}[/tex]

PLEASE ANSWER I NEED THIS NOW The blue dot is at what value on the number line?

Answers

Answer:

-10

Step-by-step explanation:

goes up by 4 each value

Answer:

-10

Step-by-step explanation:

in that chain,

it is 10,6,2,-2,-6,-10

Suppose P(x) represents the profit on the sale of x Blu-ray discs. If P(1,000) = 5,000 and P'(1,000) = −3, what do these values tell you about the profit? P(1,000) represents the profit on the sale of Blu-ray discs. P(1,000) = 5,000, so the profit on the sale of Blu-ray discs is $ . P'(x) represents the as a function of x. P'(1,000) = −3, so the profit is decreasing at the rate of $ per additional Blu-ray disc sold.

Answers

Answer:

Step-by-step explanation:

We are told that P(x)  is the profit of saling x blu ray discs. P(1000) is our profit for selling 1000 blu ray discs. So, our profit is 5000. Recall that the derivative P'(x) represents the rate at which the function P(x) is increasing/decreasing (increasing if P'(x) is positive, or decreasing otherwise) by increasing the values of x. In this case P'(1000)=-3, so the profit will decrease -3 if we increase x in one unit.

if u can do all 3 that will be great, TYSM!!

Answers

A: Five people each paying $14.50 means that the total bill was [tex]5\cdot\$14.50=\boxed{\$72.50}.[/tex]

B: Subtract the number of people he already has from the number of people he needs to get [tex]42-29=\boxed{13}[/tex] people.

C: Divide 96 by 8 to get [tex]\boxed{12}[/tex] packs.

Arithmetic is where it all begins, y'know?

Answer:

Hope this helps though I m not sure about the first one since I got another answer 72.5 but it isn't there

A 6) Set both given equations equal to zero, then combine them into one standard form
equation. Simplify if possible.
7x + 3 = 5 and y-1 = 6

Answers

The equation in standard form is given by:

[tex]7x + y - 9 = 0[/tex]

------------------

The general equation in standard form is:

[tex]Ax + By + C = 0[/tex]

------------------

The first equation is:

[tex]7x + 3 = 5[/tex]

Setting equal to zero:

[tex]7x + 3 - 5 = 0[/tex]

[tex]7x - 2 = 0[/tex]

------------------

The second equation is:

[tex]y - 1 = 6[/tex]

[tex]y - 1 - 6 = 0[/tex]

[tex]y - 7 = 0[/tex]

------------------

Combining them, and placing in standard form:

[tex]7x - 2 + y - 7 = 0 + 0[/tex]

[tex]7x + y - 9 = 0[/tex]

A similar problem is given at https://brainly.com/question/14664782

300 students in a high school freshman class are surveyed about what kinds of pets they have. Of the 300
students, 200 have a dog, 180 have a cat, and 150 have a cat and a dog. Using this information, answer
• each of the following questions.
Let D be the event that a randomly selected student has a dog and C be the event that a randomly
selected student has a cat.
What is P(D), the probability that a student in the class has a dog?
What is P(C), the probability that a student in the class has a cat?
What is P(D and C), the probability that a student in the class has a dog and a cat?
What is P(D or C), the probability that a student in the class has a dog or a cat?

Answers

Answer:

Step-by-step explanation:

What is P(D), the probability that a student in the class has a dog?

D=66%

What is P(C), the probability that a student in the class has a cat?

C=60%

What is P(D and C), the probability that a student in the class has a dog and a cat?

DC=50%

Other Questions
A bowling ball collides with a tennis ball. Which object has the larger impact force on the other considering that they have the same acceleration? Why?This is for an exam could you please answer fast The two types of jurisdiction in state courts are _____ and ____ 9(5r-2) and 14r-7 yes or no to tell if this expression is equivalent or not What is genetic drift?Please help me with this ill mark you as brainlist. On October 1, 20Y6, Jay Crowley established Affordable Realty, which completed the following transactions during the month: Oct. 1 Jay Crowley transferred cash from a personal bank account to an account to be used for the business in exchange for common stock, $40,000. 2 Paid rent on office and equipment for the month, $4,800. 3 Purchased supplies on account, $2,150. 4 Paid creditor on account, $1,100. 5 Earned sales commissions, receiving cash, $18,750. 6 Paid automobile expenses (including rental charge) for month, $1,580, and miscellaneous expenses, $800. 7 Paid office salaries, $3,500. 8 Determined that the cost of supplies used was $1,300. 9 Paid dividends, $1,500. 1. Journalize entries for transactions Oct. 1 through 9. Refer to the Chart of Accounts for exact wording of account titles. 2. Post the journal entries to the T accounts, selecting the appropriate date to the left of each amount to identify the transactions. Determine the account balances, after all posting is complete. Accounts containing only a single entry do not need a balance. 3. Prepare an unadjusted trial balance as of October 31, 20Y6. 4. Determine the following: a. Amount of total revenue recorded in the ledger. b. Amount of total expenses recorded in the ledger. c. Amount of net income for October. 5. Determine the increase or decrease in retained earnings for October. The resistivity of a metal increases slightly with increased temperature. This can be expressed as rho=rho0[1+(TT0)], where T0 is a reference temperature, usually 20C, and is the temperature coefficient of resistivity. Part A First find an expression for the current I through a wire of length L, cross-section area A, and temperature T when connected across the terminals of an ideal battery with terminal voltage V. Then, because the change in resistance is small, use the binomial approximation to simplify your expression. Your final expression should have the temperature coefficient in the numerator. Express your answer in terms of L, A, T, T0, V, rho0, and . "The Tell-Tale Heart" by Edgar Allan Poe What are three things you notice about the authors craft of The Tell-Tale Heart? (What are three things you specifically notice about his style of writing?) Please answer this correctly if you work make 15 an hour for 8 hour shift how much is that Why should government regulation hold social media companies responsible for what is posted on their platform? an olympic runner completes the 200 meter sprint in 23 seconds what is the runners avarge speed Ever since 1959 Cuba has had what type of government? Amber prefers to date men who are 20 years her senior. Although the women Jerome dates are approximately his age, they seem uncannily similar to his mother in some way. A psychoanalyst might view Amber's "Daddy" issues and Jerome's "Mommy" issues as reflecting fixation at the_____stage. find the values for a and b. Find the exact value of tan (-1740) What is the boiling point ofgold1064.43C2970C1313C3266C What are some informative speech ideas? Im supposed to present a speech and need a base topic :) BRAINLIEST ANSWER Twice the difference of a number and 4 is equal to three times the sum of the number and 6. Find the number. Whats the a chicken nugget Una pelota cae con una energa de 1 J y bota al llegar al suelo. Si tras el bote su energa es de 0,998 J, podemos afirmar que la energa se ha destruido?